Use: nCk = n!/k! (n-k)!
At a local cafeteria there are ten vegetable selections available. How many three-vegetable plates may be made

Answers

Answer 1

Answer:

120

Step-by-step explanation:

The formula [tex]nCk=\frac{n!}{k!(n-k)!}[/tex] describes how many ways [tex]k[/tex] items can be chosen given [tex]n[/tex] total items. In this scenario, [tex]n=10[/tex], and [tex]k=3[/tex], thus:

[tex]\frac{10!}{3!(10-3)!}=120[/tex]

This means that you can make 120 three-vegetable plates


Related Questions

A design for a garden is composed of a triangle and 2 rectangles. Some side lengths of the garden design are shown. 7.5 ft 12.5 ft 7.5 ft 12.5 ft What is the area of the garden in square feet?​

Answers

Answer:

215.625 square feet.

Step-by-step explanation:

The best way to approach this is by finding the area of each shape and then adding them together.

The area of a rectangle is Length · Width. Both rectangles have a length of 12.5 feet and a width of 7.5 feet. 12.5 · 7.5 = 93.75. And since both rectangles have the same dimensions, we can just multiply 93.75 by two to get 187.5 square feet for the rectangles.

The area of a triangle is Base · Height / 2. Since the sides of the triangle are the same as the sides of rectangles, the dimensions of the triangle are 7.5 feet and 7.5 feet.

7.5 · 7.5 / 2 = 28.125 square feet.

187.5 + 28.125 = 215.625 square feet.

Hope this helps! :)

please solve this question faaaast​

Answers

Answer:

300-400

Step-by-step explanation:

The first step is finding the total of the data we have. So, we take 5 + 10 + 15 + 20 + 25 + 15 + 10 which equals 80.

The median is the middle point of all the data. If it's an odd number, you can calculate the median with the equation (n+1) / 2, plugging in the total amount of data for n.

When it's an even number though, there is no one middle point since the data splits evenly in 2, so we have to use 2 equations: n/2 & (n/2) + 1. Then, we find the average of those two data points. (Although, you don't have to do that for this particular question).

When we plug 80 in for n in both of these equations, we get 40 and 41.

To where this is in the question, we have to count up from the bottom of the chart. 1-5 is below 100, 6-15 is 100-200, 16-30 is 200-300, and 31-50 is 300-400.

Since 40 and 41 are between 31 and 50, the answer is 300-400.

Hope this helps! :)

What is 3(b+8) in Distributive Property​

Answers

Answer:

3b + 24

Step-by-step explanation:

3*b=3b

3*8=24

therefore the answer is 3b+24

Hope this helps:)...if not then sorry for wasting your time and may God bless you:)

you just multiply 3 and b, it’ll give you 3b, and then 3 x 8 which gives 24. So it’ll be 3b+24

14. How much interest will you earn if depositing $25,000 to grow continuously at 9%
interest rate for 12 years?

Answers

Answer:

Step-by-step explanation:

i think this is the answer

Spencer owns a taxi cab company. He charges customers a base fee of $5 and an extra $2 for each mile traveled. If F represents a customer's fare (in dollars) and d represents the distance traveled (in miles draw a graph representing the relationship between F and d​

Answers

Answer:

Let F = customer's fare (in dollars)

Let d = distance traveled (in miles)

Given:

Base fee = $5Charge per mile = $2 per mile

Creating an equation from given information:

F = 2d + 5

This is a linear equation, with slope of 2 and y-intercept of (0, 5)

**see attached graph**

Answer:

See below ↓

Step-by-step explanation:

Given :

Base fee = $5Per extra mile = $2

Solving :

With F as the total customer fare and d is the distance covered the equation will be :-F = 2d + 5In terms of x and y, it can be written as :y = 2x + 5Graph is given below

pls help ASAP first correct answer gets brainleist ​

Answers

Answer:

4.58257569 meters (Could probably get away with rounding it to 4.6 meters)

Step-by-step explanation:

Pythag theorem

a^2 +b^2 =c^2

Insert your numbers

10^2+b^2=11^2

Simplify squares

100+b^2=121

subtract 100 from both sides

b^2=21

sqrt both sides

sqrt(b^2)=sqrt(21)

simplify sqrts to get b

b= 4.58257569

ixl math question im very lazy and i dont understand

Answers

Answer:

1/21

Step-by-step explanation:

We start with 7 cards. The probability of picking a 4 is 1/7. Since the first card is not put back, there are 6 cards left. Of these 6 cards, there are two which have divisors of 63: 3 and 7. So the probability of picking a divisor of 63 is 2/6, which reduces to 1/3. Multiplying these probabilities gives 1/21.

A circle has (-1, -1) and (-25,-11) as endpoints of a diameter.
Find the center and radius of the circle.
Write the standard equation of the circle.

Answers

well, since we know the diameter points, half-way in between is the center

[tex]~~~~~~~~~~~~\textit{middle point of 2 points } \\\\ (\stackrel{x_1}{-1}~,~\stackrel{y_1}{-1})\qquad (\stackrel{x_2}{-25}~,~\stackrel{y_2}{-11}) \qquad \left(\cfrac{ x_2 + x_1}{2}~~~ ,~~~ \cfrac{ y_2 + y_1}{2} \right) \\\\\\ \left(\cfrac{ -25 -1}{2}~~~ ,~~~ \cfrac{ -11 -1}{2} \right)\implies \left(\cfrac{-26}{2}~~,~~\cfrac{-12}{2} \right)\implies (-13~~,~~-6)[/tex]

and its radius will be half the length of the diameter

[tex]~~~~~~~~~~~~\textit{distance between 2 points} \\\\ (\stackrel{x_1}{-1}~,~\stackrel{y_1}{-1})\qquad (\stackrel{x_2}{-25}~,~\stackrel{y_2}{-11})\qquad \qquad d = \sqrt{( x_2- x_1)^2 + ( y_2- y_1)^2} \\\\\\ d=\sqrt{[-25 - (-1)]^2 + [-11 - (-1)]^2}\implies d=\sqrt{(-25+1)^2+(-11+1)^2} \\\\\\ d=\sqrt{(-24)^2+(-10)^2}\implies d=\sqrt{676}\implies d=26~\hfill \stackrel{half~that}{r=13}[/tex]

[tex]\rule{34em}{0.25pt}\\\\ \textit{equation of a circle}\\\\ (x- h)^2+(y- k)^2= r^2 \qquad center~~(\stackrel{-13}{ h},\stackrel{-6}{ k})\qquad \qquad radius=\stackrel{13}{ r} \\\\[-0.35em] ~\dotfill\\\\\ [x-(-13)]^2~~ + ~~[y-(-6)]^2~~ = ~~13^2\implies (x+13)^2~~ + ~~(y+6)^2~~ = ~~169[/tex]

what are the slope and y - intercept of the equation 6x+3y=-12

Answers

Answer:

Slope= -2

y-intercept= -4

Step-by-step explanation:

To find the slope and y-intercept of a given equation, first rearrange the equation such that it is in the slope-intercept form. In the slope-intercept form, the coefficient of y is 1 and the rest are moved to the left hand side of the equation.

[tex]\textcolor{steelblue}{\text{Slope-intercept form}}[/tex]

y= mx +c, where m is the slope and c is the y-intercept

Given equation: 6x +3y= -12

Let's leave the y term on the left of the equation and bring the rest to the right.

3y= -6x -12

To ensure that the coefficient of y is 1, divide both sides by 3.

y= -2x -4

The equation is now in the slope-intercept form.

The slope is the coefficient of x (the number that comes before x), thus the slope is -2. The y-intercept is -4.

i need help
will give 30 points

Answers

Answer:

Step-by-step explanation:

the coordinating grid shows points L, M, N, and O. all the coordinates for these points are integers. what is the value of the x-coordinates of point M

Answers

Answer:

-6

Step-by-step explanation:

Hope this helps!!!

The area of the triangle below is 3/40


square meters. What is the length of the base? Express your answer as a fraction in simplest form.

Answers

Answer:

b = 3/4

Step-by-step explanation:

The area of a triangle is given by

A = 1/2 bh

3/ 40 = 1/2 b ( 1/5)

Multiply

3/40 = 1/10 b

Multiply each side by 10

10 *3/40 = 10* 1/10 b

3/4 = b

Twenty wooden spheres, each with a radius of 2 inches, are packed snugly into a square box that is 18 inches on each side. The remaining space is filled with packing beads. What is the volume occupied by the spheres?

Vsphere=[tex]\frac{4}{3}\pi r^3[/tex]

Answers

Answer:

670.21 inches^3

Step-by-step explanation:

To find the volume of the spheres, all you need to know is the formula for the volume of a sphere, which you've included in your problem. From the problem, you also know that r = 2. From here, you calculate the volume of 1 of the spheres, which turns out to be:

V = 33.51032

Since there are 20 spheres in this box, you multiple the volume of a single sphere by 20 to get the total volume occupied by the spheres.

V = 20 * 33.51032

V = 670.2064

If you round to 2 decimal places, this becomes 670.21.

Find the value of x. Round to the nearest tenth of a unit. Show work.

Answers

Answer:

x = 41.16

Step-by-step explanation:

The value of the top angle in the triangle is equal to 90° - 27° = 63° as they are complementary anglesTo find x, take the tan ratio of the angletan63° = 1.96tan63° = x/21⇒ x/21 = 1.96⇒ x = 21 * 1.96⇒ x = 41.16

Answer:

x=41.2

Step-by-step explanation:

Assuming that the dotted line and the vertical line form a 90 degree angle, we can find the angle measurements of the triangle.

Since there is a 90 degree angle formed between the dotted line and the vertical line, subtracting 27 from 90 will get us the angle measure of one of the vertex of the triangle.

90-27=63 degrees

Now we can use trig. ratios to help us figure out the missing side.

tan(63)=x/21

x=tan(63)*21

x=41.21482062

x=41.2

Hope this helps!!!

8. Charlotte is given the equation 3x - 4y = 12. She needs to create a graph of the equation, but
isn't sure how since she is only familiar with
graphing from slope-intercept form. Devise a plan
to help Charlotte graph the equation in the space
below, and then draw your graphed line.

Answers

Answer/Step-by-step explanation:

Note:

Graphing From An Equation:

Equation are frequently written in slope-intercept form { y = mx+b}.

Which "m" represents the slope and "b" represents the y-intercept.

Solutions and Equation:

Solutions to linear equations contain any points located on the graphed line.

Ordered Pair = Solution to an equation if it's values are substituted in the equation which makes it true

Solve:

{x = 0 }       3(0) - 4y = 12

                       -4y = 12                                 (0, -3 )

                       -4y/-4 = 12/-4

                                y = -3

~~~~~~~~~~~~~~~~~~~~~~~~~~~~~~~~~~~~~~~~~~~~~~~~~~~~~~~~~~~~~~~~

{y = 0}                  3x-4(0) =12

                                3x = 12              (4,0)

                                 3x/3 = 12/3

                                        x = 4

~~~~~~~~~~~~~~~~~~~~~~~~~~~~~~~~~~~~~~~~~~~~~~~~~~~~~~~~~~~~~~~~

~Lenvy~

What fraction is equivalent to four tenths and has a denominator of 100?

Answers

Answer:

[tex]\frac{40}{100}[/tex]

Step-by-step explanation:

You can multiply on the upper and bottom part by the same number without changing the fraction so [tex]\frac{4}{10}=\frac{4\cdot 10}{10\cdot 10}=\frac{40}{100}[/tex]

Find an ordered pair (x,y) that is a solution to the equation -x+6y=1

Answers

Answer:

(5,1)

is one of infinitely many possible order pairs.

Step-by-step explanation:

There are an infinite amount of x values that give you an answer to this question. You get to choose. But first let's start off with isolating y.

[tex]-x + 6y = 1\\6y = 1 + x\\y = \frac{1+x}{6}[/tex]

Now the equation is easier to interpret. [tex]y = \frac{1+x}{6}[/tex]

Now here's the part where you actually get to choose. Take any x, value, I'll choose 5.

I'll sub it into the equation and get my y value, [tex]y = \frac{1 + (5)}{6}= 1[/tex]

Okay, we get that y = 1, when x = 5. That's an ordered pair:

(5,1)

Remember you could follow this same procedure for any x value you choose.

Mai thinks Tyler’s answer is incorrect. She says, “If expressions are equivalent then they are equal for any value of the variable. Why don’t you try to substitute the same value for in all the equations and see where they are not equal?” 1. Find the step where Tyler made an error. 2. Explain what he did wrong. 3. Correct Tyler’s work.

Answers

Mai's statement implies that Tyler's steps do not form an equivalent expression

Tyler's error is in step 3.

How to determine Tyler's error?

Tyler's steps of solving the expression is given as:

9 - 4 ( 5x - 6 )9 + (-4) ( 5x + -6 )9 + -20x + -63 - 20x

Tyler's error is in step 3.

This is so because:

9 + (-4) ( 5x + -6 ) in step 2 is not equivalent to 9 + -20x + -6 in step 3

In actual sense, Tyler's error is in multiplying the (-4) and (-6)

The correct solution is as follows:

9 - 4 ( 5x - 6 )9 + (-4) ( 5x + -6 )9 + -20x +2433 - 20x

Read more about equivalent expressions at:

https://brainly.com/question/2972832

Which inequality is shown in the graph?

Answers

Answer:

Step-by-step explanation:

If the circumference of a circle is 88 cm find its area

Answers

I hope I can help you:)

The dot plot below shows the results of a recent patient survey at Brookwood Medical Center. Use
the data to answer the following questions. If necessary, round your answer to the nearest tenth.

Find the mean and median scores.

Answers

Answer:

the median is 3.5

Step-by-step explanation:

there are 27  dots                                                                                                      27/2=13.5                                                                                                                 therefore the answer is the 13th dot and a half                                                              3.5 :)                                                                                                                                      (i dont know for sure but i think the mean is 3)

The mean is 3 and the median is 3 for this list of numbers.

From the dot pot,

Arrange the number in order:

1, 1, 2, 2, 2, 2, 3, 3, 3, 3, 3, 3, 3, 3, 3, 4, 4, 4, 4, 4, 4, 4, 4, 4, 5, 5, 5, 5, 5

Now,

Mean = Average of all the values.

Median = Midvalue of the number.

Now,

There are 30 numbers in the list, so the mean is:

Mean = 90/30 = 3

To find the median, we need to find the middle number.

Since there are an even number of numbers in the list, we take the average of the two middle numbers:

The middle two numbers are both 3, so the median is:

Median = (3 + 3)/2 = 3

Therefore,

The mean is 3 and the median is 3 for this list of numbers.

Learn more about median here:

https://brainly.com/question/28060453

#SPJ7

y= -2x - 3
y= 7x + 6 solve by elimination

Answers

Answer:

x = y = - 1

Step-by-step explanation:

y = - 2x - 3

y = 7x + 6 -----> equation 1.

So,

7x + 6 = - 2x - 3

7x + 2x = - 3 - 6

9x = - 9

Divide 9 on both sides,

9x / 9 = - 9 / 9

x = - 1

Substitute x = - 1 in equation 1,

y = 7x + 6

  = 7 ( - 1 ) + 6

  = - 7 + 6

  = 6 - 7

y = - 1

Therefore,

x = y = - 1

Step-by-step explanation:

please mark me as brainlest

In this lab, you observed how pollutants affected the ________


of water. You also modeled and observed how pollution affected freshwater sources, including surface water and the water in the _________

Answers

Answer: PH, ground

Step-by-step explanation: edge2022

In this lab, you observed how pollutants affected the" PH "of water. You also modeled and observed how pollution affected freshwater sources, including surface water and the water in the" ground "

What is water pollution?

Water pollution can be defined as the abnormal accumulation of strange elements in the water with harmful consequences to natural ecosystems.

Water pollution involves all the possible ways in which any water body can be contaminated with harmful elements.

The source of water pollution is human activities, producing the biggest damage to natural ecosystems.

Contaminating elements might be either gasses, solid material, or liquid material Such as Chemical elements, microbiota, heavy metals, and sediments affect all forms of life.

You can learn more about water pollution at

brainly.com/question/19920929

#SPJ2

Solve the system of equations at right. 2^(x+y)=16 2^(2x+y)= 1/8

Answers

Answer:

  (x, y) = (-7, 11)

Step-by-step explanation:

The system of exponential equations can be solved by the use of logarithms. Equivalently, the equations can be written so that exponents can be equated. This system is equivalent to two linear equations.

First equation:

  2^(x+y) = 16

  2^(x +y) = 2^4 . . . . use the same base to write the constant

  x +y = 4 . . . . . . . . equate exponents of 2 (take log₂ of both sides)

Second equation:

  2^(2x +y) = 1/8

  2^(2x +y) = 2^(-3) . . . . use the same base to write the constant

  2x +y = -3 . . . . . . equate exponents of 2

Solve the linear equations:

We can subtract the first equation from the second to get ...

  (2x +y) -(x +y) = (-3) -(4)

  x = -7

  y = 4 -x = 4 -(-7) = 4 +7 = 11 . . . . use the first equation to find y

The solution to the system is (x, y) = (-7, 11).

PLEASE HELP ME WITH THIS

Answers

The answer is 68 degrees

Work out m and c for the line: 2 x + 3 y + 4 = 0

Answers

Answer:

18marrsm is waiting for your help.

Add your answer and earn points.

Answer

1

jimrgrant1

Genius

30.5K answers

219.5M people helped

Answer:

m = - , c = -

Step-by-step explanation:

The equation of a line in slope- intercept form is

y = mx + c ( m is the slope and c the y- intercept )

Given

2x + 3y + 4 = 0 ( subtract 2x + 4 from both sides )

3y = - 2x - 4 ( divide all terms by 3 )

y = - x - ← in slope- intercept form

with slope m = - and y- intercept c = -

A bus makes 28 stop every 2 hours. How many stops does it make in 5 hours?

Answers

Answer:

70 stops

Step-by-step explanation:

To find the amount of stops it makes in an hour, do 28 stops ÷ 2 hours, which will give you 14 stops in one hour.

Then do 14 stops × 5 hours, which would give you 70 stops.

Here's the notation if you want to make it look pretty:

 [tex]5(\frac{28}{2} )=70[/tex]

Find the solution of the system of equations.
x+2y=9
-6x+4y=-46

Answers

Answer:

x=-3.5 and y=6.25

Step-by-step explanation:

x+2y=9

x=9-2y

-6(9-2y)+4y=46

-54+12y+4y=46

16y-54=46

16y = 100

y = 100/16 = 50/8 = 25/4 = 6.25

x+2(6.25)=9

x+12.5=9

x= -3.5

The measure of an angle is 63.9. What is the measure of its supplementary angle?

Answers

Answer:

116.1

Step-by-step explanation:

supplementary-180

complimentary-90

180-63.9= 116.1

What is the value of k?​

Answers

Answer:

10°

Step-by-step explanation:

According remote interior angles, 4k + 5 + 6k + 10 = 115.

10k + 15 = 115

10k = 100

k = 10

Hope this helps :)

Have a nice day!

Answer:

k = 10°

Step-by-step explanation:

      According to the exterior angle theorem, the exterior angle will be equal to the two opposite inside angles added together. It will look like the following, and we solve for k from there.

115° = (4k + 5)° + (6k + 10)°

115° = 4k° + 5° + 6k° + 10°

115° = 6k° + 4k° + 10°  + 5°

115° = 10k° + 15°

100° = 10k°

10° = k

k = 10°

Other Questions
Example: Yo / levantarseMe levanto.1. Ella / baarse2. Pepita y Paco / cepillarse / los dientes3. Nosotros / lavarse / las manos4. T / arreglarse / para la escuela5. Vosotros / peinarse / el peloRewrite the following sentences to show what each person should do. Use the correct forms of the verb deber with the reflexive verbs in the infinitive. Be sure to change the reflexive pronouns to match the subjects.Example: Yo / deber / ducharseDebo ducharme.6. Mam / deber / maquillarse7. Pap / deber / afeitarse8. Ellos / deber / quitarse / el pijama9. Esteban / deber / ponerse / la ropa10. Vosotros / deber / levantarseFill in the blanks in the following sentences with the correct adjectives and the correct forms of the verb estar.Example: Mara (frustrated).Mara est frustrada.11. Paco y Luis (are angry).12. Yo (am nervous).13. Beln (is tired).14. Nosotros (are busy).15. T (are anxious).Complete the following sentences by replacing the underlined portions with the correct possessive pronouns.Example: Yo tengo un libro.Es mio.16. Pablo tiene un coche. Es .17. Nosotros tenemos una casa grande. Es .18. T tienes dinero. Es .19. Vosotros tenis una televisin pequea. Es .20. Ellos tienen mucha ropa. Es what is the slope of 5x-4y=3 Find the value of y and z Liberties FRR'SRespond to all parts of the question. In your response, use substantive examples where appropriate1Following the terrorist attacks of September 11, 2001, President Bush ordered the use of wiretaps tocollect metadata on phone calls being made between the United States and countries whereterrorists were suspected of operating. Because this practice involved a very broad collection of data,few of the wiretaps were authorized by warrants.Develop an argument that takes a position on whether the collection of the data was constitutional orthat it was unconstitutional.Use at least one piece of evidence from the following foundational documents:The Fourth AmendmentThe Federalist 51The Federalist 70In your essay, you must: Respond to the prompt with a defensible claim or thesis that establishes a line of reasoning. Support your claim with at least two pieces of specific and relevant evidence.One piece of evidence must come from one of the foundational documents listed above.A second piece of evidence can come from any other foundational document not used asyour first piece of evidence, or it may be from your knowledge of course concepts. Trigonometric EquationsSolve the equation over the interval [0, 360)cos(theta) = -1/2 I need help on this question Im confused I think a &b are the same and c &d are different but Im not to sure ! Suppose we want to choose 7 letters, without replacement, from 12 distinct letters. (a) How many ways can this be done, if the order of the choices is not relevant? Use Demos Graphing CalculatorRob works part time at the Fallbrook Riding Stable. He makes $5 an hour exercisinghorses and $10 an hour cleaning stalls. Because Rob is a full-time student, he canwork no more than 12 hours per week. However, he must make at least $60 perweek.Which of the following is a possible solution for this system of inequalities?A. 1 hour exercising and 1 hour cleaning B. Two hours exercising and five hours of cleaningC. Seven hours of exercising and eight hours of cleaningD. Two hours exercising and eight hours cleaning Which effect did World War II have on the United States?a) The United States became the richest, most powerful nation in the world.b) The war caused the United States to enter another severe depression.c) The war had little effect on the United States.d) The United States became second in power only to England. A 60-g sample of radioactive isotope is measured after two half-lives have passed. What pair of percentages describes the sample after two half-lives?. dont use me for points please i need help, (circuits for physics) The grandma of a 6-year-old is asking them 2x3 what is 2x3 Why do I feel empty and numb on the inside HELPPPP!!!! what did the u.s. do to japan to end the war? was it moral or immoral? Jayden is researching edible plants and wants to learn more about the ways in which people can eat parts of the Spanish needle plant. Which is the most effective research question to accomplish this purpose Can the GCF of a pair of numbers ever equal one of the numbers? Explain how you know What did the Lord Jesus Christ say about the Scriptures in John 10:35? Use complete sentences. Write the word that has double r's and double s's.please help asap Aaron has $25 to spend at the carnival. Admission is $4.00 and the ride tickets are $1.25 each. What is the maximum number of ride tickets that Aaron can buy? Write and solve the inequality. Question 23 of 25Which ending would most likely disappoint a reader?A. An ending that surprises the readerO B. An ending the reader could easily predictO C. An ending that leaves some questions openO D. An ending with an unexpected twistI couldnt find this